Find the slope and the y-Intercept of the line.
-5x + 4y = -3
Write
your answers in simplest form.

Answers

Answer 1

Answer:

slope=5/4 y intercept=-3/4

Step-by-step explanation:

put it into slope intercept form y=5/4x-3/4

pull out 5/4 as slope and -3/4 as y intercept


Related Questions

By the triangle sum theorem, the sum of the angles in a triangle is equal to Response area. Therefore, m∠A+m∠B+m∠C=180°. Using the Substitution Property, (4x)°+90°+(x+10)°=180°. To solve for x, first combine like terms to get 5x + 100 = 180. Using the Response area, 5x = 80. Then, using the division property of equality, x = 16. To find the measure of angle A, use the Response area to get m∠A=4(16)°. Finally, simplifying the expression gets m∠A=64°.

Answers

The required measure of the angle is m∠A=64°.

What is the angle?

orientation of one line with respect to the horizontal or other respective line is known as a measure of orientation and this measure is known as the angle.

,

Here,
Angles are represented by an algebraic expression, and the sum of the angle is 180°.
According to the question,
m∠A+m∠B+m∠C=180°
Substitute the value in the equation,
4x+90°+x+10=180°
5x = 180 - 100
5x = 80
x = 16°

Thus, the required measure of the angle is m∠A=64°.

Learn more about Angles here:
https://brainly.com/question/13954458

#SPJ1

An ocicat eats $\frac 35$ of a pound of food daily. How many whole ocicats can a $19\frac 12$-pound bag of food feed for one week?

Answers

Number of ocicats that [tex]19\frac{1}{2}[/tex] food will last for one week is 4 .

In the question ,

it is given that

the amount of food that the ocicat eats in 1 day = 3/5 pound

So , the amount of food that the ocicat eats in 1 week = (3/5) × 7 = 21/5

we have to find the number of ocicat that can eat [tex]19\frac{1}{2}[/tex] pound of food feed .

So ,

the number of ocicat that can eat  [tex]19\frac{1}{2}[/tex] pound of food feed = [tex]19\frac{1}{2}[/tex] ÷ 21/5

= (19*2+1)/2 ÷ 21/5

= 39/2 × 5/21

= (39 × 5)/(2 × 21)

= 195/42

= 65/14

= [tex]4\frac{9}{14}[/tex]  ocicats

since the number of ocicats cannot be in fraction ,

so number of ocicats = 4

Therefore , Number of ocicats that [tex]19\frac{1}{2}[/tex] food will last for one week is 4 .

The given question is incomplete , the complete question is

An ocicat eats 3/5 of a pound of food daily. How many whole ocicats can a [tex]19\frac{1}{2}[/tex] -pound bag of food feed for one week?

Learn more about Division here

https://brainly.com/question/16930564

#SPJ1

help meeeeeeeee pleaseeeee rn rnnnn!!!!!!!!!!!!!!!!!!!!!!!!!!!!!!!
help meeeeeeeee pleaseeeee rn rnnnn!!!!!!!!!!!!!!!!!!!!!!!!!!!!!!!
help meeeeeeeee pleaseeeee rn rnnnn!!!!!!!!!!!!!!!!!!!!!!!!!!!!!!!
help meeeeeeeee pleaseeeee rn rnnnn!!!!!!!!!!!!!!!!!!!!!!!!!!!!!!!

Answers

The value of function is positive so the function have maximum value 30 at x=3.

In the given question we have to find the maximum or the minimum value of a function.

The given function is

f(x) = -3x^2+18x+3

To find the maximum or minimum value we firstly find the value of f'(x).

f'(x) = -6x+18

Now put f'(x)=0

-6x+18=0

Subtract 18 on both side we get

-6x=-18

Divide by -6 on both side we get

x = 3

Now finding the value of function at x=3

f(3)= -3*(3)^2+18*3+3

f(3)= -3*9+54+3

f(3)= -27+54+3

f(3)= 30

Since the value of function is positive so the function have maximum value 30 at x=3.

To learn more about maximim and minimum value of quadratic equation link is here

brainly.com/question/28769828

#SPJ1

Change the standard form equation y=x^2-4x+9 to vertex form by completing the square.

Answers

Start by finding the value of (b/2)^2 which in this case would be 4
Add 4 to the equation, maintaining balance as follows:
y=x^2-4x+4+9-4
Simplify the first 3 terms(x^2-4x+4) of the equation to
(x-2)^2 by completing the square and the last 2 terms(9-4) to 5
Your final simplified equation in vertex form by completing the square is y=(x-2)^2+5

Find x if AC= 17, BD = 2x - 6, AD= x + 16, and BC = 6

Answers

Answer:

Hey, hope it helps my friend

Step-by-step explanation:

In the image

What is the inequality of the graph below?

Answers

Answer:

it should be the first one, if not it’s the third

Step-by-step explanation:

use the following to answer the next 2 questions. solar-heat installations successfully reduce the utility bill 60% of the time. suppose 10 houses with solar-heat installations are selected at random and the outcome between houses is independent. suppose x is the number of houses that successfully reduced the utility bill out of the 10. 14. which discrete distribution will appropriately model x? explain. binomial 15. what is the probability that at least 9 out of 10 solar-heat installations are successful and will reduce the utility bill? a. 0.0464 b. 0.9432 c. 0.0403 d. 0.8429

Answers

The probability that at least 9 out of 10 solar-heat installations will succeed and reduce the utility bill is 0.0464.

What is a binomial distribution?

The probability distribution known as the binomial distribution, which is used in statistics, quantifies the chances that a value will take one of two independent values given a particular set of conditions or hypotheses.

Formula to calculate the binomial distribution

P(X = x) =ⁿCₓ× pˣ×(1-p)ⁿ⁻ˣ

Here, we know that

Solar-heat installations successfully reduce the utility bill 60% of the time.

We have to calculate the probability that at least 9 out of 10 solar-heat installations will succeed and reduce the utility bill.

We have 60% = 0.6 = p and n = 10.

We have a binomial distribution: X : (10, 0.6).

We use the binomial distribution formula to calculate the probability

P(X = x) =ⁿCₓ× pˣ×(1-p)ⁿ⁻ˣ, we get

P(x ≥ 9) = 1 - P(x < 9)

P(x ≥ 9) = 1 - P(x ≤ 8)

P(x ≥ 9) = 1 - ∑⁸₀ P(x = x)

P(X ≥ 9) = 1 - ∑⁸₀¹⁰Cₓ × (0.6)ˣ × (1-0.6)¹⁰⁻ˣ

P(X ≥ 9) = 1 -( 0.00011 + 0.00157 + 0.01062 + 0.04247 + 0.11148 + 0.20066 + 0.25082 + 0.21499 + 0.12093)

P(X ≥ 9) = 1 -0.95365

P(X ≥ 9) = 0.04635

Hence, the probability that at least 9 out of 10 solar-heat installations will succeed and reduce the utility bill is 0.0464.

To learn more about the binomial distribution from the given link

https://brainly.com/question/15223696

#SPJ4

1. Which is an example of an experiment? (1 point)
OA company asks its website users to rate its new features to navigate the site.
O Researchers working for a company test their new sneaker design to see if it helps athletes run faster.
OA scientist watches a group of local birds to see what type of tree they prefer to make their nests in.
An ice cream company asks people to identify their top three ice cream flavors.

Answers

An example of experiment is B. Researchers working for a company test their new sneaker design to see if it helps athletes run faster.

What is an experiment?

An experiment simply means a procedure carried out to support or refute a hypothesis, or determine the efficacy or likelihood of something previously untried. It should be noted that experiments provide insight into cause-and-effect as they demonstrate what outcome occurs when a particular factor is manipulated

Experiments have a vital place in the development of science because experiments are used to test the reality of a thought, an axiom, or a phenomenon with a theoretical basis and to turn it into a law or a theorem.

In this case, testing the new sneakers is an experiment.

Learn more about experiments on:

https://brainly.com/question/17274244

#SPJ1

On Friday night, Clara ate a pizza for dinner and had 1/5 of the pizza left over. On Saturday, she ate 2/3 of what was left. How much of the pizza did Clara eat on Saturday? Write your answer as a fraction or as a whole or mixed number.

Answers

Answer:

Clara ate 2/15

Step-by-step explanation:

1/5 equals 3/15

A cell phone provider offers a plan that costs ​40$ per month plus ​0.10$ per text message sent or received. A comparable plan costs ​50$ per month but offers unlimited text messaging. Complete parts a. and b. below.

a. How many text messages would have to be sent or received in order for the plans to cost the same each​ month?

In order for the plans to cost the​ same,
enter your response here text messages would have to be sent or received.
​(Simplify your answer. Type an integer or a​ decimal.)

Answers

answer:

100 text messages.

$50-$40 = $10

$10/$0.1 = 100

100 text messages

Tank B is 60% full of water. It requires an additional 2800 litres of water to be full. How many litres of water fills tank B?​

Answers

the answer is : 7,000

jake's on 49 tickets to the school are, and jeans sold 12 tickets. what is the ratio, in simplest form, of the number of tickets jeans sold to the number of tickets jake sold?

Answers

The ratio in simplest form, of the number of tickets jeans sold to the number of tickets jake sold is 12/49.

Given that,

Jake sold 49 tickets while jeans sold 12 for the school.

In simplest form the ratio of the given case will be;

Simplest form:

A fraction is a number that falls between the whole numbers but is not a whole number. It belongs to the entire. A fraction also consists of a denominator and a numerator. We obtain the fraction in its simplest form when both the numerator and denominator can no longer be independently reduced to any smaller integer.

The ratio we get;

→ 12/49

The ratio in simplest form, of the number of tickets jeans sold to the number of tickets jake sold is 12/49.

To learn more about simplest form click here:

brainly.com/question/290068

#SPJ4

the p-value is 0.026, so we reject the null hypothesis and accept the alternative hypothesis. we conclude that this year the proportion of americans who support federal spending cuts to medicaid is less than 0.12 what type of error is possible here?

Answers

The proportion of Americans who support federal spending cuts to Medicaid is less than 0.12 leads to Type 1 error.

What is Type 1 error and Type 2 error?

When we reject a true H₀ it is known as type 1 error. The probability of type 1 errors is inversely proportional to the amount of confidence you choose. There is a 5% possibility of receiving a type 1 error on a test with a 95% confidence level.

When we feel to reject a false H₀ it is known as type 2 error.

H₀:P= 0.12

Hₐ:P= 0.12

The question implies that we actually rejected the null hypothesis, while there is a chance that it is true and we are rejecting a true null hypothesis.

Therefore, it only suggests that there is a possibility that we will make a type 1 error.

Consequently, Type 1 error is the response to our query.

To know more about type 1 error and type 2 error, visit:

https://brainly.com/question/14778785

#SPJ4

which are the solutions of x2= -5x +8

Answers

x1=-6.27492, x2=1.27492
He’s right use his answer

subtract 2w+4 from 2w-6 show your work

Answers

Answer:

10

Step-by-step explanation:

(2w + 4) - (2w - 6)

2w + 4 - 2w + 6

2w - 2w + 4 + 6

10

I hope this helps!

8 + (-3) +15 - (-40)

Answers

8 - 3 + 15 + 40
8 - 58
= - 50

hope this helps !

For a ride on a rental scooter, Eric paid an $8 fee to start the scooter plus 6 cents per minute of the ride. The total bill for Eric's ride was $17.84. For how many
minutes did Eric ride the scooter?
minutes

Answers

Steps and answer in the attachment

Answer:

164 minutes

Step-by-step explanation:

$8 + $0.06m = $17.84

To isolate the m, first bring the eight to the other side

0.06m = 9.84

then divide both sides by 0.06

m = 164

The average mass of a man is 84 kg and of a woman is 70 kg
a lift can safely carry 620 kg
work out the maximum number of people the lift can safely carry

Answers

The maximum number of people the lift can safely carry is 8 people.

Given that, the average mass of a man is 84 kg and of a woman is 70 kg.

What is the average?

In maths, the average value in a set of numbers is the middle value, calculated by dividing the total of all the values by the number of values. When we need to find the average of a set of data, we add up all the values and then divide this total by the number of values.

Now, average = (84+72)/2

= 156/2

= 78

Let the number of people in the left be n.

So, 78n≤620

Divide 78 to both the sides of the inequality

78n/78≤620/78

⇒ n≤7.9

⇒ n≤8

Therefore, the maximum number of people the lift can safely carry is 8 people.

To learn more about an average visit:

https://brainly.com/question/11195029.

#SPJ1

(a) Annie and Dermot share $600 in the ratio 11:9.
(i) Show that Annie receives $330.

Answers

Ratio of amount $600 share by Annie and Dermot is 11 : 9, then amount  received by Annie is $330 and Dermot is $270.

As given in the question,

Total amount share by Annie and Dermot is $600.

Ratio of amount share by Annie and Dermot is 11 : 9

Let x be the amount of money.

11x + 9x = $600

⇒ 20x = $600

⇒20x/20 = $600/20

⇒x= $30

Amount received by Annie is equal to

= 11 × 30

= $330

Amount received by Dermot is equal to

= 9 × 30

= $270

Therefore, ratio of amount $600 share by Annie and Dermot is 11 : 9, then amount  received by Annie is $330 and Dermot is $270.

Learn more about ratio here

brainly.com/question/13419413

#SPJ1

I need help to solve this please!!​

Answers

Answer:

Adult is 10$ and Child is 7$.

Unfortunately for the family, they do not have enough to go to the theater. They do not have that extra dollar.

Hope this helps!

Step-by-step explanation:

2x + 4y = 48

5x + 2y = 64

2x + 4y = 48

10x + 4y = 128

( 10x - 2x ) + ( 4y - 4y ) = ( 128 - 48 )

8x = 80

x = 10

2 ( 10 ) + 4y = 48

20 + 4y = 48

4y = 48 - 20

4y = 28

y = 7

NEED HELP WRITING THE EQUATION!!!

Answers

The equation of line with undefined slope and passes through the point

(1/2, 5) will be;

⇒ x = 1/2

What is Equation of line?

The equation of line in point-slope form passing through the points

(x₁ , y₁) and (x₂, y₂) with slope m is defined as;

⇒ y - y₁ = m (x - x₁)

Where, m = (y₂ - y₁) / (x₂ - x₁)

Given that;

The point = (1/2 , 5)

And, The slope = undefined

Now,

Since, We know that;

The equation of line with undefined slope and passes through the point will be;

⇒ x = a

Where, 'a' is the x - intercept.

Here, The point = (1/2 , 5)

The slope = undefined

And, The x - intercept = 1/2

So, The equation of line with undefined slope and passes through the point (1/2, 5) will be;

⇒ x = 1/2

Learn more about the equation of line visit:

https://brainly.com/question/18831322

#SPJ1

Noah went into a movie theater and bought 9 drinks and 4 candies, costing a total of $60. Nachelle went into the same movie theater and bought 10 drinks and 2 candies, costing a total of $57.50. Determine the price of each drink and the price of each candy.

Answers

Answer:

See below.

Step-by-step explanation:

Let the price of drinks be x and the price of candies be y.

Condition # 1:

9x + 4y = 60 --------------------(1)

Condition # 2:

10x + 2y = 57.50 --------------(2)

Multiply Eq. (2) by 2

2(10x + 2y) = 57.50*2

20x + 4y = 115 --------------(3)

Subtract Eq. (3) from (1)

9x + 4y - 20x - 4y = 60 - 115

9x - 20x = -55

-11x = -55

11x = 55

Divide both sides by 11

x = 55/11

x = $5

Put x = 5 in Eq. (1)

9(5) + 4y = 60

45 + 4y = 60

Subtract 45 to both sides

4y = 60 - 45

4y = 15

Divide both sides by 4

y = 15/4

y = $3.75

So, the cost of one drink is $5 and the cost of one candy is $3.75.

[tex]\rule[225]{225}{2}[/tex]

Use the method of adding y-coordinates to graph the function.
f(x) = 3x - sin 4x

Answers

Graph of f(x) = 3x - sin 4x is using method of adding y-coordinates is attached below.

What is graphing method by adding ordinates?

The function y = 3x - sin 4x is actually the sum of two functions, y = 3x and y = - sin 4x.

For every value in the domain, there is a corresponding value in the range so that the point (x, 3x - sin 4x) is on the graph of y = 3x - sin 4x.

According to the given question:

Follow these steps to graphing the function:

1. Open an Excel file. Label the first column "x-coordinates," the second column "y = 3x," the third column "y = -sin 4x," and the fourth column "y = 3x - sin 4x." Set each column to round to four decimal places.

2. Set up the first column to range from -2pi to 2pi by intervals of pi/4.

3. Fill in the second and third columns by utilizing the equations y = 3x and y = - sin 4x, respectively.

4. Fill in the fourth column by simply adding the second and third columns.

5. Graph y = 3x - sin 4x  by using the x-coordinates from column A and the y-coordinates from column D.

Check the attachment below for the required graph obtained after plotting the points.

To know more about method of adding y-coordinates visit

https://brainly.com/question/27853856

#SPJ1

what is 5/2x + 1/2x = 9 + 7/2

Answers

Answer: x = 25/6

Step-by-step explanation: yes

Answer: x=25/6

Step-by-step explanation:

combine multiplied terms into 1 fraction

5/2*+1/2x=9+7/2

5x/2+1/2x=9+7/2

combine

5x/2+1/2x=9+7/2

5x/2+1x/2=9+7/2

multiply by 1

5x/2+1x/2=9+7/2

5x/2+x/2=9+7/2

Use the diagram and the given angle measure to find the other three angle measures

Answers

By using the diagram and the given angle measure we get that:

if, m∠1 = 143°

then, m∠2 = 37°

         m∠3 = 143°

         m∠4 = 37°

What is angles?

An angle is formed when two straight lines or rays meet at a common endpoint.

What is Properties of Congruence?

Reflexive Property:

       For all angles A ,

        ∠A≅∠A .

       An angle is congruent to itself.

Symmetric Property:

        For any angles A  and  B ,

        if ∠A≅∠B , then ∠B≅∠A .

         Order of congruence does not matter.

Transitive Property:

        For any angles A,B,  and  C ,

        if ∠A≅∠B and ∠B≅∠C , then ∠A≅∠C .

        If two angles are both congruent to a third angle, then the first two        angles are also congruent.

I assume that, this refers to two intersecting lines which form four angles.

∠1 and ∠2 are supplementary angles whose sum is 180°.                                      

         ∠1 + ∠2 = 180°

          143° + ∠2 = 180°

          ∠2 = 180° - 143°

          ∠2 = 37°.

∠1 and ∠3 are vertical angles so do with ∠2 and ∠4.Therefore, it is easy to find their measures because vertical angles are congruent in measure.

         It means that,  ∠1 = ∠3 =143° (properties of congruency)

         and ∠2 = ∠4 =37° (properties of congruency).

Find out more about angles and congruent angles:

https://brainly.com/question/25716982

#SPJ1

a rectangular lot is to be bounded by a fence on three sides and by a wall on the fourth side. two kinds of fencing will be used with heavy duty fencing selling for $4 a foot on the side opposite the wall. the two remaining sides will use standard fencing selling for $3 a foot. what are the dimensions of the rectangular plot of greatest area that can be fenced in at a cost of $6600?

Answers

An area 825 by 550 ft gives max area for $6600.

What is the area of rectangle?Any shape's area can be calculated by counting how many unit squares will fit inside of it.A square with a side of 1 unit is referred to as a unit square in this context. Therefore, the quantity of unit squares that make up a rectangle's perimeter is its area. Alternately, the area of the rectangle is the area contained within the boundary of this shape. The unit-length tiles in your home are a good illustration of a rectangle form. By counting the number of tiles, you can quickly determine how much space the floor takes up. This will also enable you to calculate the rectangle floor's area.

acc to our question-

he perimeterP = L + 2WThe cost4L + 3(2W) = $66004L + 6W = $66004L = (6600-6W)divide both sides by 4L = 1650 - 1.5WReplace L with (1650-1.5w)A = W(1650-1.5W)A quadratic equationA = -1.5w^2 + 1650wFind the axis of symmetry: x = -b/(2a); In this equation: a=-1.5; b=1650w = %28-1650%29%2F%282%2A-1.5%29w = +550 ft is the widthfind the lengthL = 1650 - 1.5(550)L = 825 ft is the length

hence, 3300 + 3300 = $6600

learn more about area of reactangles click here:

https://brainly.com/question/25292087

#SPJ4

3x - 2 when x=4

please be quickkkkkkkk

Answers

Answer:

10

Step-by-step explanation:

substitute x = 4 into the expression

3x - 2 = 3(4) - 2 = 12 - 2 = 10

Answer: 10

Step-by-step explanation:

4 × 3 = 12

12 - 2= 19

Hope it helps

The total weekly profit p, in dollars, from producing and selling x units of a certain product is given by the
function p(x) = 225x-(165x+c), where c is a constant. If 75 units were produced and sold last week for a profit
of $3,365, what is the value of c?
-1,135

Answers

Step-by-step explanation:

The total weekly profit p, in dollars, from producing and selling x units of a certain product is given by the function p(x) = 225x-(165x+c), where c is a constant. If 75 units were produced and sold last week for a profit of $3,365, what is the value of c?

$3,365 = 225(75) - (165(75) + c)

$3,365 = 16,875 - 165(75) - c

$3,365 = 16,875 - 12,375 - c

$3,365 = 4,500 - c

=1,135 =  - c

c = 1,135

5. (x+3)(x-3)+(x-3)²
Simplify the expression above in standard form.​

Answers

Answer: 2x^2 -6x

Step-by-step explanation:

First you should distribute the x's as shown:

(x)(x)+(x)(−3)+(3)(x)+(3)(−3)+x2+−6x+9=x2+−3x+3x+−9+x2+−6x+9

Then combine Like terms:

↓x2+−3x+3x+−9+x2+−6x+9=↓

↓(x2+x2)+(−3x+3x+−6x)+(−9+9)↓

2x2+−6x

Hope this helped... :)

Scott will purchase a new computer on an installment plan. The computer costs $2,300 and requires a 15% down payment.
a.) How much is the down payment?

Answers

The value of the down payment for the car is $345.

How to calculate the.dow payment?

From the information, Scott will purchase a new computer on an installment plan and the computer costs $2,300 and requires a 15% down payment.

The value of the down payment will be:

= Down payment percentage × Total cost

= 15% × $2300

= 0.15 × $2300

= $345

Learn more about percentages on:

brainly.com/question/24304697

#SPJ1

Other Questions
in the summer of 1941, a. philip randolph threatened to lead a massive protest on washington to pressure the government to require integration of . multiple choice question. companies with defense contracts educational institutions civil service jobs all branches of the milit chris and winsome are married and filed their taxes as married filing jointly. while filing their taxes, they determined that their total itemized deduction is $20,000. chris and winsome should briefly describe the learning features you would include in the program and discuss why you recommend including them. Find the missing side of this righttriangle.X1814X =[?]Enter the number that belongs in the green box. if mexico has a weak peso relative to the dollar, what is the impact for u.s. companies importing inputs from mexico that are priced in pesos? multiple choice the u.s. company will lose money. The green basilisk enclosure will include a rectangular pond in one corner. The width of thepond will be 3 inches less than the width, w, of the enclosure.W-3The area of the water's surface is given by this expression: w - w - 6.QuestionQuestion 1Factor the area expression, w - w - 6.Replace the values of A and B to write the expression in factored form.(w - A) (w + B) A cap 6.000 cm in diameter plugs an opening in a spherical container 1.000 m in diameter at atmospheric pressure. The container is submerged to a depth of 10.00 ft in water. Assume the plug is planar and the pressure over the container is equalized. The force needed to remove the plug is closest to? If m/AGD = 72, then what is m/DGB?17737. CAEDHa105G/13/2FB4Please provide an explanation. a nurse is caring for a client with sepsis who was recently transferred to the intensive care unit following the development of disseminated intravascular coagulation (dic). the nurse understands that dic is most likely secondary to the infection causing the release of cytokines, which can cause: ce4. Six foot tall pine trees were planted during theschool's observation of Earth Awareness Week in1990. The trees have grown at an average rate of3/4 foot per year. How tall would they be in 2010? the hyphae of arbuscular mycorrhizal fungi (amf) form bushy structures after making contact with the plasma membrane of a root cell. what is the function of these structures? Write log A as a natural logarithm. Choose the list that shows the temperatures in order from warmest to coldest.A) 11F, 12F, -11F, -12FB) 12F, 11F, -12F, -11FC) 12F, 11F, -11F, -12FD) 12F, -12F, 11F, -11F prior to recording adjusting entries, the office supplies account had a $359 debit balance. a physical count of the supplies showed $105 of unused supplies available. the required adjusting entry is: PLSSS HELP IF YOU TURLY KNOW THISS PLSS HELP!!!!!!!Tell what you know about what comparative anatomy is and how this study provides evidence for change over time. HELP!I will name you the BRAINLIEST!The thing is in the photo.I do not understand it because it looks like it has "!" Please help. Colin buys a car for 45500. It depreciates at a rate of 6% per year. How much will it be worth in 3 years? Give your answer to the nearest penny where appropriate. a rectangle has length 3 cm greater than its width. If it has an area of 28cm 2, find the dimensions of the rectangle do you think it is possible for government to outlaw everything that businesses could do wrong? if so, why does government not do that? if not, how can regulation stay ahead of rogue businesses that push the limits of the system until it breaks?